Add problems for Serway and Jewett v8's chapter 31.
[course.git] / latex / problems / Serway_and_Jewett_8 / problem31.31.tex
1 \begin{problem*}{31.31}
2 Two parallel rails with negligable resistance are $10.0\U{cm}$ apart
3 and are connected by a resistor of resistance $R_3=5.00\U{\Ohm}$.  The
4 circuit also contains two metal rods having resistances of
5 $R_1=10.0\U{\Ohm}$ and $R_2=15.0\U{\Ohm}$ sliding along the rails
6 (Fig.~P31.31).  The rods are pulled away from the resistor at constant
7 speeds of $v_1=4.00\U{m/s}$ and $v_2=2.00\U{m/s}$, respectively.  A
8 uniform magnetic field of magnitude $B=0.0100\U{T}$ is applied
9 perpendicular to the plane of the rails.  Determine the current in
10 $R_3$.
11 \begin{center}
12 % --+-----+-----+--
13 % x | x x Z x x | x
14 % <-| x x ZR3 x |->
15 % v1| x x Z x x |v2
16 % --+-----+-----+--
17 %   R1            R2
18 \begin{asy}
19 import Mechanics;
20 import ElectroMag;
21 import Circ;
22
23 real u = 1cm;
24 real w = 1mm;
25
26 MultiTerminal R3 = resistor(dir=90, "$R_3$", draw=false);
27 real rlen = R3.terminal[1].y - R3.terminal[0].y;
28 Vector B = BField(phi=-90);
29 vector_field(R3.center, width=3*u, height=rlen, v=B);
30 R3.draw();
31 real yt = R3.terminal[1].y;
32 real yb = R3.terminal[0].y;
33 wire((-1.5*u, yt), (1.5*u, yt));
34 wire((-1.5*u, yb), (1.5*u, yb));
35 dot(R3.terminal[0]);
36 dot(R3.terminal[1]);
37 pair p1 = (-u,(yt+yb)/2);
38 Vector v1 = Velocity(p1, dir=180, "$v_1$");  v1.draw();
39 Block R1 = Block(p1, width=w, height=rlen);  R1.draw();
40 label("$R_1$", (p1.x, yb), align=S);
41 pair p2 = (-p1.x, p1.y);
42 Vector v1 = Velocity(p2, "$v_2$");  v1.draw();
43 Block R2 = Block(p2, width=w, height=rlen);  R2.draw();
44 label("$R_2$", (p2.x, yb), align=S);
45 \end{asy}
46 \end{center}
47 \end{problem*}
48
49 \begin{solution}
50 \end{solution}
51